Đến nội dung

chanhquocnghiem nội dung

Có 586 mục bởi chanhquocnghiem (Tìm giới hạn từ 08-06-2020)



Sắp theo                Sắp xếp  

#737481 $\begin{vmatrix} a1 &a2 \\ b1&b2 \...

Đã gửi bởi chanhquocnghiem on 28-02-2023 - 16:30 trong Phương pháp tọa độ trong mặt phẳng

$\begin{vmatrix} a1 &a2 \\ b1&b2 \end{vmatrix}$ mọi người có hiểu này là gì không vậy 

Cái đó gọi là định thức, nó có nghĩa là $a_1b_2-a_2b_1$.
 




#736599 $\int_0^1 {f\left( x \right)dx}$

Đã gửi bởi chanhquocnghiem on 01-01-2023 - 19:01 trong Tích phân - Nguyên hàm

sao có thể tính chính xác được?

Sao lại không tính được ?

$\int_{0}^{1}f(x)dx=\int_{0}^{1}1.dx=x\Bigg|_0^1=1$.




#733469 $\left\{\begin{matrix} ax+by=(x-y)^2\...

Đã gửi bởi chanhquocnghiem on 18-05-2022 - 22:37 trong Phương trình, hệ phương trình và bất phương trình

Cho a,b,c dương. Giải hệ phương trình ẩn x,y,z

$\left\{\begin{matrix} ax+by=(x-y)^2\\by+cz=(y-z)^2 \\cz+ax=(z-x)^2 \end{matrix}\right.$

Từ đề bài ta có $\left\{\begin{matrix}ax=(x-y)(x-z)\\by=(y-z)(y-x)\\cz=(z-x)(z-y) \end{matrix}\right.$

Xét $3$ trường hợp :

1) Có ít nhất một trong ba số $x,y,z$ là số âm. Không làm mất tính tổng quát, giả sử $x< 0$.

    Khi đó $ax< 0\Rightarrow x,y,z$ là ba số phân biệt và $x$ nằm giữa $y$ và $z$ (tức là $y< x< z$ hoặc $z< x< y$)

    Không làm mất tính tổng quát, ta giả sử tiếp $y<x<z$. Khi đó $y$ cũng âm.

    Từ đó $by< 0\Rightarrow y$ nằm giữa $x$ và $z$. Như vậy trong $3$ số $x,y,z$ phân biệt có đến $2$ số nằm giữa 2 số kia.

    Điều này không thể xảy ra $\rightarrow$ trường hợp 1 vô nghiệm.

2) Có ít nhất một trong ba số $x,y,z$ bằng $0$. Giả sử $x=0$.

    Khi đó $ax=0$. Lại có 2 trường hợp nhỏ :

    a) Chỉ một trong 2 số còn lại bằng $0$, chẳng hạn $y=0$. Từ đó suy ra $z=c$

    b) Cả ba số $x,y,z$ đều bằng $0$

    Vậy trường hợp 2 có $4$ nghiệm là $(a,0,0),(0,b,0),(0,0,c)$ và $(0,0,0)$

3) Cả $x,y,z$ đều dương $\rightarrow ax> 0,by> 0,cz> 0\rightarrow x,y,z$ là 3 số dương phân biệt.

    Vì $ax> 0\Rightarrow x$ không nằm giữa $y$ và $z$

    Tương tự, $by> 0\Rightarrow y$ không nằm giữa $x$ và $z$

    Và $cz> 0\Rightarrow z$ không nằm giữa $x$ và $y$

    Vậy là trong 3 số phân biệt không có số nào nằm giữa cả. Điều vô lý này $\rightarrow$ trường hợp 3 vô nghiệm.

 

Kết luận : Bài toán có $4$ nghiệm là $(a,0,0),(0,b,0),(0,0,c)$ và $(0,0,0)$.
 




#733524 $\left\{\begin{matrix} ax+by=(x-y)^2\...

Đã gửi bởi chanhquocnghiem on 24-05-2022 - 17:39 trong Phương trình, hệ phương trình và bất phương trình

Giải hệ phương trình $\left\{\begin{matrix}ax+by=(x-y)^2\\by+cz=(y-z)^2\\cz+ax=(z-x)^2 \end{matrix}\right.$

với $a,b,c$ là các số thực bất kỳ.

 

------------------------------------------------------------------------

Ta xét các trường hợp sau :

Trường hợp 1 : $a=b=c=0$

Dễ thấy trường hợp này có nghiệm $(x,y,z)=(t,t,t), \forall t\in \mathbb{R}$

Trường hợp 2 : $a,b,c$ là số thực tùy ý thỏa mãn $\frac{1}{a}+\frac{1}{b}+\frac{1}{c}\neq 0$ (kể cả trường hợp có $1$ hoặc $2$ số bằng $0$)

Trường hợp này có tối đa $4$ nghiệm như bạn Nesbit đã giải ở trên.

Trường hợp 3 : $a,b,c$ là các số thực thỏa mãn $\frac{1}{a}+\frac{1}{b}+\frac{1}{c}=0$

$\frac{1}{a}+\frac{1}{b}+\frac{1}{c}=0\Rightarrow ab+bc+ca=0\Rightarrow ab^2+cb^2=ac^2+bc^2\Rightarrow \frac{a+b}{a+c}=\frac{b^2}{c^2}$

Từ hệ phương trình đã cho suy ra : $\left\{\begin{matrix}y^2-(2x+b)y+x^2-ax=0\\z^2-(2x+c)z+x^2-ax=0\\by+cz=(y-z)^2 \end{matrix}\right.$

Xét 2 trường hợp nhỏ :

  1) $a+b> 0,a+c> 0$

    Khi đó, với mỗi $t$ khác $0$ và thỏa mãn $t> -\frac{b^2}{4(a+b)}=-\frac{c^2}{4(a+c)}$, ta có $2$ nghiệm.

    a) Nếu $bc< 0$, $2$ nghiệm đó là

       $\left\{\begin{matrix}x=t\\y=\frac{2t+b+\sqrt{4(a+b)t+b^2}}{2}\\z=\frac{2t+c+\sqrt{4(a+c)t+c^2}}{2} \end{matrix}\right.$ và $\left\{\begin{matrix}x=t\\y=\frac{2t+b-\sqrt{4(a+b)t+b^2}}{2}\\z=\frac{2t+c-\sqrt{4(a+c)t+c^2}}{2} \end{matrix}\right.$

    b) Nếu $bc> 0$, $2$ nghiệm đó là

       $\left\{\begin{matrix}x=t\\y=\frac{2t+b+\sqrt{4(a+b)t+b^2}}{2}\\z=\frac{2t+c-\sqrt{4(a+c)t+c^2}}{2} \end{matrix}\right.$ và $\left\{\begin{matrix}x=t\\y=\frac{2t+b-\sqrt{4(a+b)t+b^2}}{2}\\z=\frac{2t+c+\sqrt{4(a+c)t+c^2}}{2} \end{matrix}\right.$

   Còn với $t=0$, ta có $3$ nghiệm $(0,b,0)$, $(0,0,c)$ và $(0,0,0)$

   Và với $t=-\frac{b^2}{4(a+b)}$, ta có nghiệm duy nhất $\left ( -\frac{b^2}{4(a+b)};\frac{b(2a+b)}{4(a+b)};\frac{c(2a+c)}{4(a+c)} \right )$

  2) $a+b< 0,a+c< 0$

     Khi đó, với mỗi $t$ khác $0$ và thỏa mãn $t< -\frac{b^2}{4(a+b)}=-\frac{c^2}{4(a+c)}$, ta có $2$ nghiệm.

    a) Nếu $bc< 0$, $2$ nghiệm đó là

       $\left\{\begin{matrix}x=t\\y=\frac{2t+b+\sqrt{4(a+b)t+b^2}}{2}\\z=\frac{2t+c+\sqrt{4(a+c)t+c^2}}{2} \end{matrix}\right.$ và $\left\{\begin{matrix}x=t\\y=\frac{2t+b-\sqrt{4(a+b)t+b^2}}{2}\\z=\frac{2t+c-\sqrt{4(a+c)t+c^2}}{2} \end{matrix}\right.$

    b) Nếu $bc> 0$, $2$ nghiệm đó là

       $\left\{\begin{matrix}x=t\\y=\frac{2t+b+\sqrt{4(a+b)t+b^2}}{2}\\z=\frac{2t+c-\sqrt{4(a+c)t+c^2}}{2} \end{matrix}\right.$ và $\left\{\begin{matrix}x=t\\y=\frac{2t+b-\sqrt{4(a+b)t+b^2}}{2}\\z=\frac{2t+c+\sqrt{4(a+c)t+c^2}}{2} \end{matrix}\right.$

     Còn với $t=0$, ta có $3$ nghiệm $(0,b,0)$, $(0,0,c)$ và $(0,0,0)$

     Và với $t=-\frac{b^2}{4(a+b)}$, ta có nghiệm duy nhất $\left ( -\frac{b^2}{4(a+b)};\frac{b(2a+b)}{4(a+b)};\frac{c(2a+c)}{4(a+c)} \right )$

Như vậy, ở trường hợp 1 và trường hợp 3, chúng ta có vô số nghiệm như đã chỉ rõ ở trên.




#738723 $\sum_{k=1}^n\left(\lfloor k\varphi\r...

Đã gửi bởi chanhquocnghiem on 20-04-2023 - 11:21 trong Dãy số - Giới hạn

Gợi ý: Chứng minh với $m$ nguyên dương thì
\begin{align*}\left\lfloor m\varphi\right\rfloor-\left\lfloor\dfrac{m}{\varphi}\right\rfloor&=m\\
\left\lfloor m\varphi\right\rfloor+\left\lfloor\dfrac{m}{\varphi+1}\right\rfloor&=2m-1
\end{align*}

$(2)\Leftrightarrow \left \lfloor m\varphi \right \rfloor-m=\left \lfloor \frac{m}{\varphi } \right \rfloor$

        $\Leftrightarrow \left \lfloor \frac{m+m\sqrt{5}}{2} \right \rfloor-m=\left \lfloor \frac{2m}{\sqrt{5}+1} \right \rfloor$

        $\Leftrightarrow \left \lfloor \frac{m\sqrt{5}-m}{2} \right \rfloor=\left \lfloor \frac{2m}{\sqrt{5}+1} \right \rfloor$

        $\Leftrightarrow \left \lfloor \frac{m(\sqrt{5}-1)}{2} \right \rfloor=\left \lfloor \frac{2m}{\sqrt{5}+1} \right \rfloor$ (cái này là hiển nhiên)




#738727 $\sum_{k=1}^n\left(\lfloor k\varphi\r...

Đã gửi bởi chanhquocnghiem on 20-04-2023 - 14:20 trong Dãy số - Giới hạn

Gợi ý: Chứng minh với $m$ nguyên dương thì
\begin{align}\left\lfloor m\varphi\right\rfloor+\left\lfloor\dfrac{m}{\varphi+1}\right\rfloor&=2m-1
\end{align}

Để chứng minh cái này ta sẽ sử dụng mệnh đề sau :

Mệnh đề
Nếu $a\notin \mathbb{Z}$ thì $\left \lfloor a \right \rfloor+\left \lfloor -a \right \rfloor=-1$.

(Chứng minh mệnh đề trên không có gì khó khăn)

Bây giờ ta chứng minh mệnh đề $(3)$ ở trên.

$\left \lfloor m\varphi \right \rfloor=\left \lfloor \frac{m+m\sqrt{5}}{2} \right \rfloor$

$\left \lfloor \frac{m}{\varphi+1 } \right \rfloor=\left \lfloor \frac{2m}{3+\sqrt{5}} \right \rfloor=\left \lfloor \frac{(3-\sqrt{5})m}{2} \right \rfloor=\left \lfloor \frac{3m-m\sqrt{5}}{2} \right \rfloor$

+ Nếu $m$ chẵn ($m=2k$)

   $\left \lfloor \frac{m+m\sqrt{5}}{2} \right \rfloor+\left \lfloor \frac{3m-m\sqrt{5}}{2} \right \rfloor=k+3k+\left \lfloor \frac{m\sqrt{5}}{2} \right \rfloor+\left \lfloor \frac{-m\sqrt{5}}{2} \right \rfloor=4k-1=2m-1$

+ Nếu $m$ lẻ ($m=2k+1$)

   $\left \lfloor \frac{m+m\sqrt{5}}{2} \right \rfloor+\left \lfloor \frac{3m-m\sqrt{5}}{2} \right \rfloor=\left \lfloor \frac{2k+1+(2k+1)\sqrt{5}}{2} \right \rfloor+\left \lfloor \frac{6k+4-1-(2k+1)\sqrt{5}}{2} \right \rfloor$

$=k+3k+2+\left \lfloor \frac{1+(2k+1)\sqrt{5}}{2} \right \rfloor+\left \lfloor \frac{-1-(2k+1)\sqrt{5}}{2} \right \rfloor=4k+2-1=2m-1$
 




#738729 $\sum_{k=1}^n\left(\lfloor k\varphi\r...

Đã gửi bởi chanhquocnghiem on 20-04-2023 - 14:40 trong Dãy số - Giới hạn

Chứng minh rằng:
\begin{equation}\label{e1}
\sum_{k=1}^n\left(\left\lfloor k\varphi\right\rfloor +\left\lfloor\frac{k}{\varphi+1}\right\rfloor\right)=n^2
\end{equation}
Trong đó $\varphi=\frac{1+\sqrt 5}{2}$ là tỉ lệ vàng.

Áp dụng kết quả trên, ta có :

$\sum_{k=1}^{n}\left ( \left \lfloor k\varphi \right \rfloor+\left \lfloor \frac{k}{\varphi +1} \right \rfloor \right )=\sum_{k=1}^{n}(2k-1)=2.\frac{n(n+1)}{2}-n=n^2$.
 




#729672 $A\subseteq X, B\subseteq X, A\ne B$ và $A...

Đã gửi bởi chanhquocnghiem on 13-08-2021 - 17:06 trong Số học

Cho tập $X=\{1,2,3,\cdots,10\}$. Tính số các cặp $\{A,B\}$ thoả mãn: $\left\{\begin{matrix}A\subseteq X, B\subseteq X, A\ne B \\ A\cap B=\{2,3,5,7\}\end{matrix}\right.$
 

$A$ và $B$ đều chứa các phần tử $2,3,5,7$.

Còn mỗi phần tử $1,4,6,8,9,10$ chỉ có $3$ khả năng :

- Thuộc $A$ (không thuộc $B$)

- Thuộc $B$ (không thuộc $A$)

- Không thuộc $A$, cũng không thuộc $B$

 

Suy ra có tất cả $3^6=729$ cặp tập hợp $\{A,B\}$ thỏa mãn yêu cầu đề bài.




#725279 $a^{a}+1;a^{a^{a}}+1;a^{a^{a^{a}}}+1;...$

Đã gửi bởi chanhquocnghiem on 12-04-2021 - 09:40 trong Các dạng toán THPT khác

CMR nếu $a$ là số tự nhiên chẵn thì mỗi số hạng của dãy số $a^{a}+1;a^{a^{a}}+1;a^{a^{a^{a}}}+1;...$ đều không chia hết cho một số lẻ bất kì

Thử chọn $a=2$ :

$a^a+1=5$ (chia hết cho $5$)

$a^{a^a}+1=17$ (chia hết cho $17$)

................................

................................

Và nếu chọn "số lẻ bất kỳ" là $1$ thì sao nhỉ ??? ---> đề sai !
 




#727728 $b^3\equiv 111\pmod{125}$ suy ra $b\e...

Đã gửi bởi chanhquocnghiem on 30-05-2021 - 11:39 trong Số học

 

Cho số tự nhiên $b$ thoả mãn:

$$b^3\equiv 111\pmod{125}.$$
Khi đó: $b\equiv ?\pmod{125}$.

$b^3\equiv 111\pmod{125}\Rightarrow \left[\begin{array}{l}b^3\equiv 11\pmod{100}\\b^3\equiv 61\pmod{100}\\b^3\equiv 36\pmod{100}\\b^3\equiv 86\pmod{100} \end{array}\right.\Rightarrow \left[\begin{array}{l}b\equiv 1\pmod{10}\\b\equiv 6\pmod{10} \end{array}\right.$

Ta chỉ cần xét trong phạm vi $b\in\left [ 0;124 \right ]$. Xét 2 trường hợp :

+ $b=10k+1$ : Khi đó $b^3=(10k+1)^3=1000k^3+300k^2+30k+1\Rightarrow b^3\equiv 30k+1\pmod{100}$

   $\left[\begin{array}{l}b^3\equiv 11\pmod{100}\\b^3\equiv 61\pmod{100} \end{array}\right.\Rightarrow \left[\begin{array}{l}k=2\\k=12\\k=7 \end{array}\right.$

   Thử lại $21^3=9261\not\equiv 111\pmod{125}$ (loại) ; $121^3=1771561\not\equiv 111\pmod{125}$ (loại) ; $71^3=357911\not\equiv 111\pmod{125}$ (loại)

 

+ $b=10k+6$ : Khi đó $b^3=(10k+6)^3=1000k^3+1800k^2+1080k+216\Rightarrow b^3\equiv 80k+16\pmod{100}$

   $\left[\begin{array}{l}b^3\equiv 36\pmod{100}\\b^3\equiv 86\pmod{100} \end{array}\right.\Rightarrow \left[\begin{array}{l}k=4\\k=9 \end{array}\right.$

   Thử lại $46^3=97336\not\equiv 111\pmod{125}$ (loại) ; $96^3=884736\equiv 111\pmod{125}$ (thỏa mãn)

 

Vậy $b\equiv 96\pmod{125}$.
 




#739100 $e^{\sin x}-\cos x$ và $\sqrt{x+...

Đã gửi bởi chanhquocnghiem on 08-05-2023 - 09:43 trong Giải tích

 

Dạ câu 2 đúng rồi ạ ! Em cảm ơn.

Với cho em hỏi: Hàm $v(x)=\sqrt{x+\sqrt{x}}$ có tập xác định là $D=[0,+\infty)$ thì làm sao tồn tại giới hạn $\displaystyle \lim_{x \to 0} v(x)=0$ được ạ ?

Đúng ra phải là $\displaystyle \lim_{x \to 0^+} v(x)=0$ chứ nhỉ ? 
Như thế thì $v$ có phải là VCB khi $x \to 0$ không ?

 

Vâng, mình nhầm, đã sửa lại.
 




#739096 $e^{\sin x}-\cos x$ và $\sqrt{x+...

Đã gửi bởi chanhquocnghiem on 08-05-2023 - 07:15 trong Giải tích

Cho các hàm số: $u(x)=e^{\sin x}-\cos x$ và $v(x)=\sqrt{x+\sqrt{x}}$

1. Hỏi $u$ và $v$ có phải là các VCB khi $x \to 0$ không ?

2. Tính giới hạn: $L=\displaystyle \lim_{x \to 0^+} \dfrac{u(x)}{v(x)}$.

1) $\lim_{x\to 0}u(x)=\lim_{x\to 0}(e^{\sin x}-\cos x)=0$

    $\lim_{x\to 0^+}v(x)=\lim_{x\to 0^+}\sqrt{x+\sqrt{x}}=0$

    $\lim_{x\to 0}v(x)$ không tồn tại.

    Vậy $u$ là VCB khi $x\to 0$, còn $v$ thì không ($v$ chỉ là VCB khi $x\to 0^+$)

2) Áp dụng quy tắc L'Hospital :

    $\lim_{x \to 0^+} \dfrac{u(x)}{v(x)}=\frac{\lim_{x \to 0^+} u'(x)}{\lim_{x \to 0^+} v'(x)}$

    $\lim_{x \to 0^+} u'(x)=\lim_{x \to 0^+}(e^{\sin x}\cos x+\sin x)=1$

    $\lim_{x \to 0^+} v'(x)=\lim_{x \to 0^+}\left ( \frac{1+\frac{1}{2\sqrt{x}}}{2\sqrt{x+\sqrt{x}}} \right )=\lim_{x \to 0^+}\left ( \frac{2\sqrt{x}+1}{4\sqrt{x^2+x\sqrt{x}}} \right )=+\infty$

    $\Rightarrow \lim_{x \to 0^+} \dfrac{u(x)}{v(x)}=0$.

   
 




#737321 $f(tan2x) = tan^4x + \dfrac{1}{tan^4x} \fo...

Đã gửi bởi chanhquocnghiem on 18-02-2023 - 17:32 trong Phương trình hàm

Không biết bài này có lm theo đơn ánh,toàn ánh,song ánh đc ko ta :??

 

Đặt $t=tan2x$

 

Khi đó $t = \dfrac{2tanx}{1-tan^2x}$

 

$\to \dfrac{1}{t} = \dfrac{1}{2tanx} - \dfrac{tanx}{2}$

 

$\to \dfrac{2}{t} = \dfrac{1}{tanx} - tanx$

 

$\to \dfrac{4}{t^2} = \dfrac{1}{tan^2x} + tan^2x - 2$

 

$\to \dfrac{4}{t^2} + 2 = \dfrac{1}{tan^2x} + tan^2x $

 

$\to \dfrac{16}{t^4} + \dfrac{16}{t^2} + 4 = \dfrac{1}{tan^4x} + tan^4x + 2$

 

$\to \dfrac{16}{t^4} + \dfrac{16}{t^2} + 2 = \dfrac{1}{tan^4x} + tan^4x $

 

Vậy hàm cần tìm có dạng : $f(t) = \dfrac{16}{t^4} + \dfrac{16}{t^2} + 2$

 

Còn cái BĐT kia ai hd vs ạ  ~O)  :ukliam2:

Vậy cần phải chứng minh $\frac{16}{\sin^4x}+\frac{16}{\cos^4x}+\frac{16}{\sin^2x}+\frac{16}{\cos^2x}\geqslant 192$
$\Leftrightarrow \frac{16(\sin^4x+\cos^4x)}{\sin^4x\cos^4x}+\frac{16}{\sin^2x\cos^2x}\geqslant 192$

$\Leftrightarrow \sin^4x+\cos^4x+\sin^2x\cos^2x\geqslant 12\ \sin^4x\cos^4x$

$\Leftrightarrow 1-\sin^2x\cos^2x\geqslant 12\ \sin^4x\cos^4x$

$\Leftrightarrow 1\geqslant \frac{1}{4}\ \sin^22x+\frac{3}{4}\ \sin^42x$

Bất đẳng thức cuối cùng là hiển nhiên.




#729284 $|x-1| + |x-2| + ... + |x-100| = 101x - 200$

Đã gửi bởi chanhquocnghiem on 01-08-2021 - 10:12 trong Phương trình, hệ phương trình và bất phương trình

Giải phương trình:

$|x-1| + |x-2| + ... + |x-100| = 101x - 200$

Xét các trường hợp :

1) $x< 1$ :

    $(1-x)+(2-x)+...+(100-x)=101x-200\Leftrightarrow x=\frac{1750}{67}$ (loại)

2) $x\geqslant 100$ :

    $(x-1)+(x-2)+...+(x-100)=101x-200\Leftrightarrow x=-4850$ (loại)

3) $1\leqslant x< 100$ :

    Đặt $k\leqslant x< k+1$ (với $k$ là số nguyên từ $1$ đến $99$)

    $(x-1)+(x-2)+...+(x-k)+(k+1-x)+(k+2-x)+...+(100-x)=101x-200$

    $\Leftrightarrow kx-(100-k)x-\frac{k(k+1)}{2}+\frac{(k+101)(100-k)}{2}=101x-200\Leftrightarrow x=\frac{k^2+k-5250}{2k-201}$

    với $k$ phải thỏa mãn $k\leqslant \frac{k^2+k-5250}{2k-201}< k+1\Leftrightarrow k^2-202k+5250\geqslant 0> k^2-200k+5049\Leftrightarrow k=30$

    Vậy $x=\frac{k^2+k-5250}{2k-201}=\frac{1440}{47}$ (thỏa mãn)

 

Kết luận : Nghiệm duy nhất là $x=\frac{1440}{47}$.
 




#727385 $|z|$ ? biết: $2(z-i)^{2021}=(\sqrt{3...

Đã gửi bởi chanhquocnghiem on 21-05-2021 - 11:01 trong Tổ hợp - Xác suất và thống kê - Số phức

Gọi $z$ là số phức thoả mãn:

\[ 2(z-i)^{2021}=(\sqrt{3}+i)(iz-1)^{2021}. \]

Xác định modul của $z$.

Đặt $z=a+bi$ ($a,b\in\mathbb{R}$). Ta có : $2\left [ a+(b-1)i \right ]^{2021}=(\sqrt3+i)(-b-1+ai)^{2021}$

$\Rightarrow 2\left ( \sqrt{a^2+(b-1)^2} \right )^{2021}=2\left ( \sqrt{(b+1)^2+a^2} \right )^{2021}\Rightarrow b=0$.

Vậy $z$ là số thực ($z=a$) và ta có : $2(a-i)^{2021}=(\sqrt3+i)(ai-1)^{2021}$

$\Rightarrow \arg\left ( 2(a-i)^{2021} \right )=\arg\left ( (\sqrt3+i)(ai-1)^{2021} \right )$  $(^*)$

Đặt $\arg(a+i)=\alpha$ ($\alpha \in(0;\pi)$), ta có :

$\arg(a-i)=-\alpha$  ; $\arg(\sqrt3+i)=\frac{\pi}{6}$  ; $\arg(ai-1)=\frac{\pi}{2}+\alpha$

Nên từ $(^*)$ ta được $-2021\ \alpha +k.2\pi=\frac{\pi}{6}+2021\left ( \frac{\pi}{2}+\alpha \right )$

$\Rightarrow 4042\ \alpha +\frac{2\pi}{3}=k.2\pi$

$\Rightarrow \alpha =\frac{(3k-1)\pi}{6063}$ ($k\in\mathbb{N},1\leqslant k\leqslant 2021$)

$\Rightarrow a=\cot\alpha$ và $|z|=|a|=\left | \cot\alpha \right |=\left | \cot\frac{(3k-1)\pi}{6063} \right |$ ($k\in\mathbb{N},1\leqslant k\leqslant 2021$)

(Nghĩa là có tất cả $2021$ giá trị $|z|$ khác nhau thỏa mãn điều kiện đề bài)

 




#736300 ...có tổng các phần tử là bội số của 3

Đã gửi bởi chanhquocnghiem on 17-12-2022 - 08:15 trong Tổ hợp - Xác suất và thống kê - Số phức

3/ Có bao nhiêu tập con 6 phần tử của $\left\{ 0,1,2,3,...,9\right \}$ có tổng các phần tử là bội số của 3.

Đặt $A=\left \{ 0,3,6,9 \right \}$ ; $B=\left \{ 1,4,7 \right \}$ ; $C=\left \{ 2,5,8 \right \}$.

$\textbf{TH1}$ (Có $1$ tập không đóng góp phần tử nào, $2$ tập còn lại, mỗi tập đóng góp $3$ phần tử)

Có $C_4^3C_3^3+C_4^3C_3^3+C_3^3C_3^3=9$ tập con.

$\textbf{TH2}$ (Cả $3$ tập đều có đóng góp phần tử)

Có $C_4^4C_3^1C_3^1+C_4^2C_3^2C_3^2=63$ tập con.

Đáp án là $9+63=72$ tập con thỏa mãn.
 




#736303 ...có tổng các phần tử là bội số của 3

Đã gửi bởi chanhquocnghiem on 17-12-2022 - 10:51 trong Tổ hợp - Xác suất và thống kê - Số phức

1/ Có bao nhiêu số tự nhiên có 8 chữ số mà các chữ số này xuất hiện ít nhất 2 lần.

$\textbf{TH1}$ (chỉ có $1$ loại chữ số, tức là $8$ chữ số giống nhau) : $9$ số.

$\textbf{TH2}$ (có $2$ loại chữ số)

Có $(C_9^1C_8^2C_8^1+C_9^1C_7^1+C_9^1C_7^5)+(C_9^1C_8^3C_8^1+C_9^1C_7^2+C_9^1C_7^4)+(C_9^2C_8^4+C_9^1C_7^3)=9639$

$\textbf{TH3}$ (có $3$ loại chữ số)

Có $(C_9^1C_8^4C_8^2C_4^2+C_7^4C_9^2C_4^2+C_7^2C_9^1C_6^2C_8^1)+(C_9^1C_8^2C_8^2C_6^3+C_7^2C_9^2C_6^3+C_7^3C_9^1C_5^2C_8^1)=317520$

$\textbf{TH4}$ (có $4$ loại chữ số)

Có $C_9^4C_8^2C_6^2C_4^2+C_7^2C_9^3C_6^2C_4^2=476280$

Vậy số số tự nhiên thỏa mãn yêu cầu đề bài là $9+9639+317520+476280=803448$.
 




#736293 ...có tổng các phần tử là bội số của 3

Đã gửi bởi chanhquocnghiem on 16-12-2022 - 22:57 trong Tổ hợp - Xác suất và thống kê - Số phức

2/ Có bao nhiêu tập con của $\left\{ 1,2,3,...,300 \right \}$ có tổng các phần tử là bội số của 3.

Gọi $A=\left \{ 3,6,9,...,300 \right \}$ ; $B=\left \{ 1,4,7,...,298 \right \}$ ; $C=\left \{ 2,5,8,...,299 \right \}$

Một tập con thỏa mãn điều kiện đề bài sẽ gồm $m$ phần tử thuộc $A$, $3n+q$ phần tử thuộc $B$ và $3p+q$ phần tử thuộc $C$

($0\leqslant m\leqslant 100$ ; $0\leqslant n,p\leqslant 33$ ; $0\leqslant q\leqslant 2$)

$\textbf{TH1}$ ($q=0$)

+ Chọn $m$ phần tử thuộc $A$ : $2^{100}$ cách.

+ Chọn $3n$ phần tử thuộc $B$ : $C_{100}^0+C_{100}^3+C_{100}^6+...+C_{100}^{99}=\frac{2^{100}-1}{3}$ cách

+ Chọn $3p$ phần tử thuộc $C$ : $C_{100}^0+C_{100}^3+C_{100}^6+...+C_{100}^{99}=\frac{2^{100}-1}{3}$ cách

$\textbf{TH2}$ ($q=1$)

+ Chọn $m$ phần tử thuộc $A$ : $2^{100}$ cách.

+ Chọn $3n+1$ phần tử thuộc $B$ : $C_{100}^1+C_{100}^4+C_{100}^7+...+C_{100}^{100}=\frac{2^{100}-1}{3}$ cách

+ Chọn $3p+1$ phần tử thuộc $C$ : $C_{100}^1+C_{100}^4+C_{100}^7+...+C_{100}^{100}=\frac{2^{100}-1}{3}$ cách

$\textbf{TH3}$ ($q=2$)

+ Chọn $m$ phần tử thuộc $A$ : $2^{100}$ cách.

+ Chọn $3n+2$ phần tử thuộc $B$ : $C_{100}^2+C_{100}^5+C_{100}^8+...+C_{100}^{98}=\frac{2^{100}+2}{3}$ cách

+ Chọn $3p+2$ phần tử thuộc $C$ : $C_{100}^2+C_{100}^5+C_{100}^8+...+C_{100}^{98}=\frac{2^{100}+2}{3}$ cách

 

Số tập con thỏa mãn yêu cầu đề bài là :

$2^{100}\left [ 2\left ( \frac{2^{100}-1}{3} \right )^2+\left ( \frac{2^{100}+2}{3} \right )^2 \right ]=2^{100}\left ( \frac{2^{200}+2}{3} \right )$.




#736142 ...không nhiều hơn tổng số bi ở 3 hộp kia.

Đã gửi bởi chanhquocnghiem on 09-12-2022 - 09:14 trong Tổ hợp - Xác suất và thống kê - Số phức

1/ Có bao nhiêu cách bỏ 14 viên bi giống nhau vào 4 hộp khác nhau sao cho số bi ở hộp cuối cùng không nhiều hơn tổng số bi ở 3 hộp kia.

Ta có hàm sinh $f(x)=(1+x+x^2+...+x^{14})^3=\left ( \frac{1-x^{15}}{1-x} \right )^3=(1-3x^{15}+3x^{30}-x^{45})\sum_{k=0}^{\infty}C_{k+2}^2x^k$

Với $k$ từ $7$ đến $14$ ta có $\left [ x^k \right ]f(x)=C_{k+2}^2$

Vậy số cách là $\sum_{k=7}^{14}C_{k+2}^2=C_9^2+C_{10}^2+...+C_{16}^2=C_{17}^3-C_9^3=596$.




#736140 ...không nhiều hơn tổng số bi ở 3 hộp kia.

Đã gửi bởi chanhquocnghiem on 09-12-2022 - 07:50 trong Tổ hợp - Xác suất và thống kê - Số phức

1/ Có bao nhiêu cách bỏ 14 viên bi giống nhau vào 4 hộp khác nhau sao cho số bi ở hộp cuối cùng không nhiều hơn tổng số bi ở 3 hộp kia.

Gọi số bi ở hộp cuối cùng là $k$ ($0\leqslant k\leqslant 7$) $\Rightarrow x+y+z=14-k\rightarrow$ Có $C_{16-k}^2$ nghiệm nguyên không âm

Vậy số cách là $C_9^2+C_{10}^2+C_{11}^2+...+C_{16}^2=C_{17}^3-C_9^3=596$.




#736149 ...không nhiều hơn tổng số bi ở 3 hộp kia.

Đã gửi bởi chanhquocnghiem on 09-12-2022 - 14:04 trong Tổ hợp - Xác suất và thống kê - Số phức

1/Em tính phần bù. Gọi $z_i$ là số bi trong hộp $i$ và đặt $z^{'}=z_4-z_1-z_2-z_3\geq 1$ thì :
$\begin {cases}
z_1+z_2+z_3+z_4&=14\\
z_1+z_2+z_3 \geq z_4;z_i\geq 0
\end{cases} \Longrightarrow \begin {cases}
2z_1+2z_2+2z_3+z^{'}&=14\\
z_1,z_2,z_3 \geq 0; \\
z^{'}= z_4-z_1-z_2-z_3 \geq 1
\end{cases}$
Ta có hàm sinh :
$f(z)=\frac{1}{(1-z^2)^3}\frac{z}{1-z}$ suy ra hàm sinh cho số cách bỏ thỏa yêu cầu là :
$g(z)=\frac{1}{(1-z)^4}-\frac{1}{(1-z^2)^3}\frac{z}{1-z}$
Vậy số cách bỏ thỏa yêu cầu là :
$\Longrightarrow \left[z^{14}\right]g(z)=596$

Nếu $z'=z_4-z_1-z_2-z_3$ thì $z'\leqslant 0$ chứ ?
 




#739979 [Biến tấu] Có bao nhiêu cách bỏ 5 bi đỏ, 5 bi xanh, 5 bi vàng vào 3 hộp khác...

Đã gửi bởi chanhquocnghiem on 11-06-2023 - 20:27 trong Tổ hợp - Xác suất và thống kê - Số phức

Nếu sử dụng hàm sinh thì ...

$f(x,y,z)=\left [ (x+x^2+...+x^5)(y+y^2+...+y^5)(z+z^2+...+z^5) \right ]^3=$

    $=x^3y^3z^3\left ( \frac{1-x^5}{1-x}.\frac{1-y^5}{1-y}.\frac{1-z^5}{1-z} \right )^3$

Số cách là $\left [ x^5y^5z^5 \right ]f(x,y,z)=216$.
 




#739959 [Biến tấu] Có bao nhiêu cách bỏ 5 bi đỏ, 5 bi xanh, 5 bi vàng vào 3 hộp khác...

Đã gửi bởi chanhquocnghiem on 11-06-2023 - 08:48 trong Tổ hợp - Xác suất và thống kê - Số phức

[Biến tấu 2] Có bao nhiêu cách bỏ 5 bi đỏ, 5 bi xanh, 5 bi vàng vào 3 hộp khác nhau sao cho mỗi hộp có ít nhất 1 viên bi, biết rằng các viên bi chỉ khác nhau về màu sắc.

Thiết lập hàm sinh :

$f(x,y,z)=\left [ (1+x+x^2+...+x^5)(1+y+y^2+...+y^5)(1+z+z^2+...+z^5)-1 \right ]^3$

      $=\left (\frac{1-x^6}{1-x}.\frac{1-y^6}{1-y}.\frac{1-z^6}{1-z}-1 \right )^3=\sum_{k=0}^{3}(-1)^kC_3^k\frac{(1-x^6)^{3-k}}{(1-x)^{3-k}}.\frac{(1-y^6)^{3-k}}{(1-y)^{3-k}}.\frac{(1-z^6)^{3-k}}{(1-z)^{3-k}}$

Số cách là $\left [ x^5y^5z^5 \right ]f(x,y,z)=8616$.
 




#739957 [Biến tấu] Có bao nhiêu cách bỏ 5 bi đỏ, 5 bi xanh, 5 bi vàng vào 3 hộp khác...

Đã gửi bởi chanhquocnghiem on 11-06-2023 - 08:30 trong Tổ hợp - Xác suất và thống kê - Số phức

[Biến tấu 1] Có bao nhiêu cách bỏ 5 bi đỏ, 5 bi xanh, 5 bi vàng vào 3 hộp khác nhau sao cho hộp nào cũng có đủ 3 loại bi, biết rằng các viên bi chỉ khác nhau về màu sắc.

Số cách chính là số bộ nghiệm nguyên dương của hệ $3$ phương trình, $9$ ẩn dưới đây

$\left\{\begin{matrix}d_1+d_2+d_3=5\\x_1+x_2+x_3=5\\v_1+v_2+v_3=5 \end{matrix}\right.$

và bằng $\left ( C_4^2 \right )^3=216$ cách.




#729804 [Hỏi] có công thức nào tương tự nhị thức newton để triển khai $a^{n...

Đã gửi bởi chanhquocnghiem on 18-08-2021 - 16:51 trong Phương trình - hệ phương trình - bất phương trình

Mọi người cho em hỏi là có công thức nào tương tự nhị thức newton để triển khai $a^{n}-b^{n}$ với n là số hữu tỉ không ạ. 

Công thức tổng quát $X^k-Y^k=\left ( X-Y \right )\left ( X^{k-1}+X^{k-2}Y+...+Y^{k-1} \right )$

1) $n=0$ (điều kiện $a,b$ khác $0$) : Ta có $a^n-b^n=a^0-b^0=1-1=0$

2) $n$ nguyên dương : Áp dụng CTTQ với $X=a$ ; $Y=b$ ; $k=n$

3) $n$ nguyên âm (điều kiện $a,b$ khác $0$) :

    Đặt $n=-m$ ($m$ nguyên dương)

    $a^n-b^n=a^{-m}-b^{-m}=\left ( \frac{1}{a} \right )^m-\left ( \frac{1}{b} \right )^m$

    Áp dụng CTTQ với $X=\frac{1}{a}$ ; $Y=\frac{1}{b}$ ; $k=m$

4) $n$ dương, không nguyên (điều kiện $a,b$ dương)

    Đặt $n=\frac{p}{q}$ ($p,q$ nguyên dương)

    $a^{\frac{p}{q}}-b^{\frac{p}{q}}=\left ( \sqrt[q]{a} \right )^p-\left ( \sqrt[q]{b} \right )^p$

    Áp dụng CTTQ với $X=\sqrt[q]{a}$ ; $Y=\sqrt[q]{b}$ ; $k=p$

5) $n$ âm, không nguyên (điều kiện $a,b$ dương)
    Đặt $n=-\frac{p}{q}$ ($p,q$ nguyên dương)

    $a^{-\frac{p}{q}}-b^{-\frac{p}{q}}=\left ( \sqrt[q]{\frac{1}{a}} \right )^p-\left ( \sqrt[q]{\frac{1}{b}} \right )^p$

    Áp dụng CTTQ với $X=\sqrt[q]{\frac{1}{a}}$ ; $Y=\sqrt[q]{\frac{1}{b}}$ ; $k=p$